Which of the following transformations shifts all points graphed in the standard (wy)
coordinate plane down 5 coordinate units?
A. (X, y-5)
B. (X, y+5)
C. (X-5y)
D. (K-5, y)
E. (X+5,y)

Answers

Answer 1

Answer:

B is your answer

Step-by-step explanation:


Related Questions

Plz help no links I will give brainiest to whoever helps

Answers

Answer:

110.92

Step-by-step explanation:

Assuming that the parameters (P), (h), and (B) all represent dimensions of the given prism, then based on the given information, the following can be concluded:

P = 6.6

h = 4.5

B = 2.2

The surface area is the two-dimensional area around a three-dimensional surface. In other words, if one was going to wrap the figure, the surface area is the amount of paper one would need. One can find the surface area by finding the area of each individual side and then adding all the results together. To find the area of a 2-dimensional figure by multiplying the length by the width.

(4.5) * (6.8) = 30.6

(4.5) * (6.8) = 30.6

(2.2) * (6.8) = 14.96

(2.2) * (6.8) = 14.96

(4.5) * (2.2) = 9.9

(4.5) * (2.2) = 9.9

Now add up all of the values,

30.6 + 30.6 + 14.96 + 14.96 + 9.9 + 9.9

= 110.92

Is it possible for a matrix to have the vector (3, 1, 2) in its row space and (2, 1, 1)T in its null space? Ex- plain.
Let a; be a nonzero column vector of an m x n matrix A. Is it possible for a j, to be in N(AT)? Explain.

Answers

It is not possible for a matrix to have the vector (3, 1, 2) in its row space and (2, 1, 1)T in its null space. Let's explain why.

Let A be an m × n matrix, and let x be a nonzero vector in the null space of A, so Ax = 0. We can also say that x is in the null space of A transpose. So x is an element of N(AT).Let’s prove the contradiction that arises from the initial claim by assuming that 3,1,2 is a row vector in the row space of A and 2,1,1 is a column vector in N(AT).We have that A[3 1 2]T = 0 and 2,1,1 is in the null space of A transpose. We also know that if a vector v is in the row space of A, then there exists a vector y such that v = A*y, where y is a column vector. So in this case, we can say that 3,1,2 is in the row space of A if there is a column vector y such that A * y = [3 1 2]T. But if that's the case, then we have the following equation: A* y = [3 1 2]. This can be written as: TA* = [3 1 2]If we then take the transpose of both sides, we have: A* y = [3 1 2]T and TA = [3 1 2]. However, this implies that TA* = TA, which can only be true if A is a symmetric matrix. But A is an m × n matrix, where m and n are not equal, so A cannot be a symmetric matrix. Therefore, it is not possible for a matrix to have the vector (3, 1, 2) in its row space and (2, 1, 1)T in its null space.

To know more about transpose, click here:

https://brainly.com/question/2263930

#SPJ11

Prove the following using a proof by contradiction:

The average of four real numbers is greater than or equal to at least one of the numbers.

Answers

Our assumption that the average of four real numbers is less than all of the numbers is false. By contradiction, we conclude that the average of four real numbers is greater than or equal to at least one of the numbers.

To prove the statement using a proof by contradiction, we assume the opposite, namely, that the average of four real numbers is less than all of the numbers. Let's denote the four numbers as a, b, c, and d. We assume that the average of these numbers, which we'll denote as avg, is less than a, b, c, and d.

Now, let's consider the sum of these four numbers: a + b + c + d. The average of these numbers, avg, is calculated by dividing the sum by 4. Therefore, we have avg = (a + b + c + d)/4.

If avg is less than a, b, c, and d, then (a + b + c + d)/4 < a, (a + b + c + d)/4 < b, (a + b + c + d)/4 < c, and (a + b + c + d)/4 < d.

Now, let's consider the sum of these inequalities: (a + b + c + d)/4 + (a + b + c + d)/4 + (a + b + c + d)/4 + (a + b + c + d)/4 < a + b + c + d.

Simplifying the left-hand side, we have (a + b + c + d) + (a + b + c + d) + (a + b + c + d) + (a + b + c + d) < 4(a + b + c + d).

This simplifies to 4(a + b + c + d) < 4(a + b + c + d), which is a contradiction. The left-hand side is greater than the right-hand side, which contradicts our initial assumption.

Therefore, our assumption that the average of four real numbers is less than all of the numbers is false. By contradiction, we conclude that the average of four real numbers is greater than or equal to at least one of the numbers.

Know more about Contradiction here:

https://brainly.com/question/28568952

#SPJ11

Malik and Nora are playing a video game.

Malik starts with m points and Nora starts n points.

Then Malik gets 150 more points, while Nora loses 50 points.

Finally, Nora gets a bonus and her score is doubled.

Nora now has 50 more points than Malik.

Enter an equation that represents the relationship between m and n

given the information above.

Answers

Answer:

Equation below

Step-by-step explanation:

An equation that represents the relationship between m and n is 2(n - 150) - (m + 150) = 50 .

The expression that represents Malik's score after he gets 150 points = m + 150

The expression that represents Nora's score after she loses 50 points = n - 150

Nora's score after her score is doubled = 2(n - 150)

The difference between Nora and Malik's score is 50. This can be represented as:  2(n - 150) - (m + 150) = 50

To learn more about equations, please check: https://brainly.com/question/16244955

Denira needs to run 9 4/10 miles this week to meet her goal for her training plan. So far this week she has run 3 1/2 miles on Monday and 2 1/2 miles on Tuesday. How many more miles does she need to run this week in order to meet her goal

Answers

Answer:

3 2/5

Step-by-step explanation:

Add the distance she already ran, and subtract the sum from the total she needs to run.

Add two distances she ran:

3 1/2 + 2 1/2 = 3 + 2 + 1/2 + 1/2 = 5 + 1 = 6

Subtract sum from total:

9 4/10 - 6 = 3 4/10 = 3 2/5

Answer:

She needs to run 3 4/10 more miles

Step-by-step explanation:

If you add the amount she ran on Monday and the amount she ran on Tuesday you get 6 miles then subtract the 6 miles minus 9 4/10 you will get 3 4/10.

Bon Air Elementary School has 300 students. The principal of the school thinks that the average IQ of students at Bon Air is at least 110. To prove her point, she administers an IQ test to 20 randomly selected students.

Among the sampled students, the average IQ is 108 with a standard deviation of 10. Based on these results, should the principal accept or reject her original hypothesis? Assume a significance level of 0.01.

Answers

The principal should reject her original hypothesis, as the lower bound of the interval is less than 110.

What is a t-distribution confidence interval?

The t-distribution is used when the standard deviation for the population is not known, and the bounds of the confidence interval are given according to the equation presented as follows:

[tex]\overline{x} \pm t\frac{s}{\sqrt{n}}[/tex]

The variables of the equation are listed as follows:

[tex]\overline{x}[/tex] is the sample mean.t is the critical value.n is the sample size.s is the standard deviation for the sample.

The critical value, using a t-distribution calculator, for a two-tailed 90% confidence interval, with 20 - 1 = 19 df, is t = 2.8609.

The parameters for this problem are given as follows:

[tex]\overline{x} = 108, s = 10, n = 20[/tex]

Then the lower bound of the interval is given as follows:

[tex]108 - 2.8609 \times \frac{10}{\sqrt{20}} = 101.6[/tex]

The upper bound of the interval is given as follows:

[tex]108 + 2.8609 \times \frac{10}{\sqrt{20}} = 114.4[/tex]

More can be learned about the t-distribution at https://brainly.com/question/17469144

#SPJ4

If You Have NO EXPLANATION Don't ANSWER

Answers

Answer:

C

Step-by-step explanation:

y=5x reads "y equals (5 times x)", which we can rephrase to "the value of y is 5 times the value of x"

y=x+5 reads "y equals (x plus 5)", which we can rephrase to "the value of y is 5 more than the value of x".

Ergo, answer C is what we're looking for.

Answer: C

Step-by-step explanation:

in y=5x, we can see a 5 placed in front of x. If there is no addition, subtraction, or division sign between a number and a variable, it always means it's multiplication.

We know now that this is 5 times x.

In y=x+5, we see that 5 is being added to x. Therefore, y is 5 more than x.

So there you have it.

What is the description of angle 4 as it relates to the situation below?
angle 4 is the angle of elevation from the person to the radar tower.

angle 4 is the angle of depression from the radar tower to the person.

angle 4 is the angle of depression from the person to the radar tower.

angle 4 is the angle of elevation from the radar tower to the person.

Answers

In the given situation, "angle 4 is the angle of elevation from the radar tower to the person" is the description of angle 4.In trigonometry, an angle of elevation or inclination is the angle between the horizontal and the line of sight of an observer looking upwards. An angle of depression is the angle between the horizontal and the line of sight of an observer looking downwards.

In the given situation, angle 4 refers to the angle formed between the horizontal and the line of sight from the radar tower to the person. As the angle is formed while looking upwards from the radar tower to the person, it is called the angle of elevation. Hence, the correct description of angle 4 in this situation is "angle 4 is the angle of elevation from the radar tower to the person."

Know more about angle of elevation:

https://brainly.com/question/29008290

#SPJ11

Can pls someone help with my homework pls I need help

Answers

Answer:

[tex] \sin(m < q) = \frac{7}{9} \\ \sin(m < q) =(0.77777777778) \\ m < q = { \sin 0.77777777778}^{ - 1} \\ m < q = (51.05755873102)[/tex]

Test the claim that the proportion of people who own cats is significantly different than 70% at the 0.02 significance level. The null and alternative hypothesis would be: 0.7 Hop 0.7 Hop - 0.7 H:P < 0.7 HP >0.7 HP 0.7 HOP The test is: right tailed left-tailed two-tailed Based on a sample of 500 people, 62% wned cats The p-value is:

Answers

Test the claim that the proportion of people who own cats is significantly different than 70% at the 0.02 significance level. The p-value is 0.024.

The null and alternative hypotheses for the claim that the proportion of people who own cats is significantly different than 70% at the 0.02 significance level are:

H0: p = 0.7 (null hypothesis

)H1: p ≠ 0.7 (alternative hypothesis)

The test is a two-tailed test because the alternative hypothesis includes not equal to (<>) which means either p is less than 0.7 or greater than 0.7

Based on a sample of 500 people, 62% owned cats.

This means that the sample proportion, p = 0.62.

To calculate the p-value, we will use the z-test statistic.

The formula for calculating the z-test statistic is given as:

z = (p - P) / √(PQ/n) where P is the hypothesized proportion (P = 0.7), Q is the complement of P (Q = 1 - P), and n is the sample size.

Using the given values in the formula, we have; z = (0.62 - 0.7) / √(0.7 × 0.3 / 500) = -2.52

The p-value for a two-tailed test at 0.02 level of significance is obtained from the standard normal table.

The area in both tails beyond the z-score of 2.52 is 0.012.

Therefore, the p-value is:

p-value = 2 × 0.012 = 0.024

To learn more about p-value

https://brainly.com/question/13786078

#SPJ11

Which line is the best model for the data in the scatter plot?

PLEASE GIVE THE CORRECT ANSWER AND FAST

Answers

honesty i don’t know i just need the points report and i’ll tell

Answer:

Upper right corner.

Step-by-step explanation:

I took the test and that one was right. Hope this helps!

Simplify (8y6)
what’s the answer?

Answers

The answer is 48 you basically just do 8 times 6 if I’m right.

Choose the equation that best describes the situation below.
Lee is 32 years younger than his mother and his mother is 67 years old. How old is Lee?
a = Lee's age

Answers

Answer:

67-32=a

if his mom is 67 and he is 32 years younger 67 minus 32 would equal a which is his age

A wardrobe has 3 pants , 5 shirts , and 7 ties .

The number of total possible outfits is 15 .

True

False

Answers

A wardrobe with 3 pants, 5 shirts, and 7 ties, has a possible outcome of 105 outfits and not 15. So the answer is False

False. The number of total possible outfits is not 15. To calculate the number of possible outfits, we need to multiply the number of choices for each item together. In this case, we have 3 choices for pants, 5 choices for shirts, and 7 choices for ties. Therefore, the total number of possible outfits would be 3 x 5 x 7 = 105.

The statement incorrectly states that there are only 15 possible outfits. It's important to consider that when selecting multiple items, the total number of combinations is found by multiplying the number of choices for each item together. In this scenario, with 3 pants, 5 shirts, and 7 ties, there are 105 possible outfits, not 15.

Learn more about Combination:

https://brainly.com/question/28065038

#SPJ4


[tex]12 \frac{3}{6} + 14\frac{4}{6} [/tex]
i don't know what's the answer i been trying it this but i can't​

Answers

27 1/6 or 163/6 or 27.16

Answer:

27 1/6

Step-by-step explanation:

14 + 12=26

4+3=7

26 7/6= 27 1/6

at a meeting ,everyone shakes hands exactly once with every other person . if there are 55 handshakes . then what Is the number of people attending​

Answers

11.
Every handshake takes 2. If no one shakes hands twice with another, there are 10+1 total people shaking hands (11).

Name them 1 thru 11. Make a line and 11 goes down the line and shakes 10 hands and then leaves. 10 then shakes 9 hands and leaves. 8 then shakes 7 hands and leaves. Etc. 2 shakes hands with 1 and leaves. 1 just leaves.

11— 10 hs

10— 9 hs

9— 8 hs

8— 7 hs

7— 6 hs

6— 5 hs

5— 4 hs

4— 3 hs

3— 2 hs

2— 1 hs

1— leaves

Total handshakes 55, total people 11.

Please help me, i'm so confused

Answers

9514 1404 393

Answer:

  (d)  512√3

Step-by-step explanation:

The area (A) of a regular hexagon can be computed from its side length (s) using the formula ...

  A = (3√3)/2·s²

Here, the side length is given as ...

  s = 32√3/3 = 32/√3

Then the area is ...

  [tex]A=\dfrac{3\sqrt{3}}{2}\cdot\left(\dfrac{32}{\sqrt{3}}\right)^2=\dfrac{3\sqrt{3}\cdot1024}{2\cdot3}=\boxed{512\sqrt{3}}[/tex]

The area of the hexagon is 512√3 square units.

CAN SOMEONE HELP ME!!!!

Thion Drones is a newly established manufacturer of drones for recreational use. The firm produced 180 drones last month and sold these for an average price of $230. Thion Drones had average variable costs of $190 per drone. Its fixed costs per month are $4,500.

a.  Calculate the average fixed cost (AFC) for Thion Drones.

   

b.  Calculate the monthly profit or loss made by Thion Drones.

Answers

what is the probability if you randomly select comedy book replace it and another comedy book?
(if your going to answer pls explain )

The average fixed cost of each case will be $25

What is the average fixed cost ?Fixed cost is the cost that does not change with the number of lawyers hired or the number of cases. Fixed cost remains fixed regardless of the number of lawyers or the number of cases. Examples of fixed cost include rent, electricity.Average fixed cost is the total fixed cost per case. Average fixed cost can be determined by dividing the fixed cost by the number of cases.

Average fixed cost = fixed cost / number of cases.

$4500/ 180= $25

To learn more about average fixed cost refer

https://brainly.com/question/28503168

#SPJ2

Use elimination to solve for x and y:
9x - 2y = 46
x + 2y = 14

Answers

Answer:

(6, 4 )

Step-by-step explanation:

Given the 2 equations

9x - 2y = 46 → (1)

x + 2y = 14 → (2)

Adding the 2 equations term by term will eliminate the y- term

10x + 0 = 60

10x = 60 ( divide both sides by 10 )

x = 6

Substitute x = 6 into either of the 2 equations and solve for y

Substituting into (2)

6 + 2y = 14 ( subtract 6 from both sides )

2y = 8 ( divide both sides by 2 )

y = 4

solution is (6, 4 )

The answer 3h - 5 < 13?

Answers

Answer:  h < 6

Step-by-step explanation:

3h - 5 < 13

3h < 18

h < 6

Answer:

h<6

Step-by-step explanation:

Eleven increased by three times a number equals 68) Write an equation for this situation and then find the
number

Answers

Answer:

11+3x=68

x=19

Step-by-step explanation:

11+3x=68 is your equation.

subtract 11 from both side to get 3x alone

3x=68-11

3x=57

divide 3 from both sides to get x alone

x=57/3

x=19

19 is your number.

We wish to estimate what percent of adult residents in a certain county are parents. Out of 200 adult residents sampled, 166 had kids. Based on this, construct a 90% confidence interval for the proportion, p. of adult residents who are parents in this county. Give your answers as decimals, to three places.
_________________ < p < _________________

Answers

Based on this, a 90% confidence interval for the proportion, p. of adult residents who are parents in this county is 0.787 < p < 0.873.

The point estimate for the proportion is calculated by dividing the number of adults with kids by the total sample size. In this case, the point estimate is 166/200 = 0.83.

To construct the confidence interval, we can use the formula:

[tex]p \pm z \times \sqrt{\frac{p \times (1 - p )}{n}}[/tex]

Where:

p is the point estimate

z is the z-score corresponding to the desired confidence level (90% confidence corresponds to a z-score of approximately 1.645)

n is the sample size

Substituting the values into the formula, we get:

[tex]= 0.83 \pm 1.645 \times \sqrt{\frac{0.83 \times (1-0.83)}{200}}[/tex]

Calculating the values, we can obtain the 90% confidence interval for the proportion of adult residents who are parents.

To construct a 90% confidence interval for the proportion of adult residents who are parents in the county, we can use the sample proportion and the standard error formula. Out of the 200 adult residents sampled, 166 had kids.

we calculate the sample proportion, p-hat:

p-hat = 166 / 200

         = 0.83

Next, we calculate the standard error using the formula:

SE = √((p-hat × (1 - p-hat)) / n)

SE = √((0.83 × (1 - 0.83)) / 200) ≈ 0.025

To construct the confidence interval, we use the formula:

p-hat ± (z × SE)

where, z is the z-score corresponding to the desired confidence level.

For a 90% confidence interval, the z-score is approximately 1.645.

Substituting the values into the formula, we get:

= 0.83 ± (1.645 × 0.025)

Calculating the upper and lower bounds:

Lower bound = 0.83 - (1.645 × 0.025) ≈ 0.787

Upper bound = 0.83 + (1.645 × 0.025) ≈ 0.873

Therefore, the 90% confidence interval for the proportion of adult residents who are parents in the county is approximately 0.787 < p < 0.873.

Learn more about confidence intervals here: brainly.com/question/32546207

#SPJ11

Mancini's Pizzeria sells four types of pizza crust. Last week, the owner tracked the number sold of each type, and this is what he found. Type of Crust Number Sold Thin crust 312 Thick crust 245 Stuffed crust 179 Pan style 304

Answers

Question:

Mancini's Pizzeria sells four types of pizza crust. Last week, the owner tracked the number sold of each type, and this is what he found.

Type of Crust Number Sold

Thin crust          312

Thick crust          245

Stuffed crust          179

Pan style                  304

Based on this information, of the next  4500 pizzas he sells, how many should he expect to be thick crust? Round your answer to the nearest whole number. Do not round any intermediate calculations.

Answer:

1060 thick crusts

Step-by-step explanation:

Given

The above table

Required

Expected number of thick crust for the next 4500

For last week data, calculate the proportion of thick crust sold

[tex]\hat p = \frac{Thick\ crust}{Total}[/tex]

[tex]\hat p = \frac{245}{312+245+179+304}[/tex]

[tex]\hat p = \frac{245}{1040}[/tex]

[tex]\hat p = 0.235577[/tex]

For the next 4500;

[tex]n = 4500[/tex]

The expected number of thick crust is (E(x)):

[tex]E(x) = \hat p * n[/tex]

[tex]E(x) = 0.235577 * 4500[/tex]

[tex]E(x) = 1060.0965[/tex]

[tex]E(x) \approx 1060[/tex]

Elena prepared 8 kilograms of dough after working 2 hours. How much dough did Elena prepare if she worked for 9 hours? Assume the relationship is directly proportional.

Answers

the answer should be 7

Answer:

36 kilograms

Step-by-step explanation:

Since she made 8 kilograms of dough over the span of 2 hours, you divide 8 by 2 and get 4 then you have to multiply 9 hours by 4 kilograms of dough to get 36 kilograms of dough.


4 people can dig a trench in 3 hours.
How long would it take 9 people?
Give your answer in minutes.

Answers

Answer:

80 minutes.

Step-by-step explanation:

do I need to explain? I hate explaining :(

Let a < b. If ƒ is continuous on [a, b], and ƒ(a) = f(b), then there there exists c € (a,b) such that f'(c) = 0. d) If f is differentiable on (0, 1), then f is uniformly continuous on (0,1).

Answers

Yes, if ƒ is differentiable on (0, 1), then ƒ is uniformly continuous on (0, 1).

In mathematics, the concept of differentiability plays a crucial role in understanding the behavior of functions. If a function ƒ is differentiable on the interval (0, 1), it means that the derivative ƒ'(x) exists for every point x in that interval.

The answer states that if a function is differentiable on (0, 1), then it is uniformly continuous on the same interval.

To understand this result, we need to consider the properties of differentiability and uniform continuity.

Differentiability implies that the function has a well-defined tangent line at every point within the interval. This implies that the function cannot exhibit abrupt changes or discontinuities, as it must be smooth and continuous.

Uniform continuity, on the other hand, deals with the behavior of a function as the input values get arbitrarily close to each other. It ensures that the function does not exhibit extreme fluctuations or rapid oscillations.

If a function is differentiable on (0, 1), then it satisfies the conditions required for uniform continuity. This is because the derivative of the function acts as a measure of its rate of change.

If the derivative is bounded (i.e., it does not become infinitely large or small), then the function can be guaranteed to be uniformly continuous.

Learn more about differentiability

brainly.com/question/13958985

#SPJ11

Please help me with the question please ASAP

Answers

Answer:

The ratio of perimeter of ABCD to perimeter of WXYZ = [tex]\frac{2}{3}[/tex]

Step-by-step explanation:

First, we have to determine the multiplicative factor of the dimensions for both figures.

Considering sides AB and WX,

multiplicative factor = [tex]\frac{12}{8}[/tex]

                                = 1.5

So that:

XY = 6 x 1.5 = 9

YZ = 7 x 1.5 = 10.5

ZW = 7 x 1.5 = 10.5

Perimeter of ABCD = 6 + 7 + 7 + 8

                                = 28

Perimeter of WXYZ = 9 + 10.5 + 10.5 + 12

                                 = 42

The ratio of the perimeters of the two quadrilaterals can be determined as;

ratio = [tex]\frac{perimeter of ABCD}{Perirmeter of WXYZ}[/tex]

       = [tex]\frac{28}{42}[/tex]

       = [tex]\frac{2}{3}[/tex]

The ratio of the perimeter of ABCD to perimeter of WXYZ is [tex]\frac{2}{3}[/tex].

A system of equations consists of two lines. One line passes through (8,4) and (6.3) and the second line passes through (0, -2) and (4.0).

Answers

Answer:

system is:

y = 1/2x

y = 1/2x - 2

No Solution to this system

Step-by-step explanation:

You guy's will get 40 points if you help me!

Answers

Answer:

mean = 5+9+9+6+6+11+8+4/7 = 8.29

median = 6

mode = 6

range = 11 - 4 = 7

Answer:

Step-by-step explanation:

5 , 9 , 6 , 6 , 11 , 8 , 4

Mean = sum of all data ÷ number of data

         [tex]= \frac{5+9+6+6+11+8+4}{7}\\\\= \frac{49}{7}\\\\= 7[/tex]

Median: To find median, arrange in ascending order and medianis the middle term

4 , 5 , 6 , 6 , 8 , 9 , 11

Middle term = 4th term

Median = 6

Mode: a number that appears most often is mode

6 appears 2 times

Mode = 6

Range:

Range = maximum value - minimum value

          = 11 - 4

          = 7

A normal distribution has a mean u = 67.3 and a standard deviation of o=9.3 Find P81, which separates the bottom 81% from the top 19%.

Answers

Value of x corresponding to P81 is 59.06.

A normal distribution has a mean u = 67.3 and a standard deviation of o=9.3.

The task is to find P81, which separates the bottom 81% from the top 19%.

For any normally distributed variable z with mean u and standard deviation o, the cumulative distribution function is defined as the probability of a standard normal variable being less than or equal to z.

A standard normal distribution has a mean of 0 and a standard deviation of 1.

That is, the variable z can be calculated as: z = (x - u) / o

The value P(z < z0) can be read off a standard normal table for any value z0.

As the normal distribution is symmetric, we can use the fact that P(z < -z0) = 1 - P(z < z0).

We now calculate z as follows: z0 = (P81 + 1) / 2 = 0.9051

From a standard normal table, we can see that P(z < 0.9051) = 0.8186.

Therefore, P(z < -0.9051) = 1 - P(z < 0.9051) = 0.1814.

Now we calculate the corresponding value of x:

z = (x - u) / o-0.9051 = (x - 67.3) / 9.3x = 59.06

Therefore, P81 corresponds to the value x = 59.06.

To learn more about normal distribution

https://brainly.com/question/12892403

#SPJ11

Other Questions
Are the lines in the diagram perpendicular, parallel, skew, or none of these?l and m:l and n:m and n:Are the lines in the diagram perpendicular, parallel, skew, or none of these?l and m:l and n: A student is writing a proof that if n >1 and n is a natural number then n3 n is divisible by 6. Write down, using an equation, (meaning use an equals sign what the inductive hypothesis would be. Do NOT write a full proof, just write the inductive hypothesis, P(k).) A supertrain of proper length 205 m travels at a speed of 0.86c as it passes through a tunnel having proper length 74 m. How much longer is the tunnel than the train or vice versa as seen by an observer at rest with respect to the tunnel? riley operates a plumbing business and this year the 3-year old van he used in the business was destroyed in a traffic accident. the van was originally purchased for $23,400 and the adjusted basis was $5,375 at the time of the accident. although the van was worth $6,340 at the time of accident, insurance only paid riley $1,625 for the loss. what is the amount of riley's casualty loss deduction? economy with a government but with no foreign trade? A. Household saving will be equal to investment B. The Sum of consumption and saving will be equal to the sum of income and taxes. C. The sum of household income and business profits must be equal to the sum of consumption, government spending, and investment D. An increase in the government deficit will force an increase in investment. E. If the government has a balanced budget, household consumption will equal the difference between income and saving. a 3.10 kg grinding wheel is in the form of a solid cylinder of radius 0.100 m. .What constant torque will bring it from rest to an angular speed of 1200 rev/min in 2.5 s? When examining distributions of numerical data, what three components should you look for? a. Symmetry, skewness, and spread b. Shape, symmetry, and spread c. Symmetry, center, and spread d. Shape, center, and spread A Democrat finds an established test to measure attitudes about public transportation, and 27 randomly selected subjects are given the test. The sample mean score is 76.2 and the standard deviation is 21.4 Using the sample data above, construct a 95% confidence Interval for the mean of the population of all such subjects piaget would argue that as an adolescent, mildred is better able to understand calculus because she is in the sensorimotor stage.a. trueb. false A 250 - resistor is connected in series with a 4.80 - F capacitor. The voltage across the capacitor is vC=(7.60V)sin[(120rad/s) t ].Derive an expression for the voltage VR across the resistor. Hayden bikes 1.8 miles in 6 minutes. His friend Jordan bikes 3.2 miles in 8 minutes. Part A: Who bikes at a faster speed? Explain your answer. scarcity is intensified when wants are limited and people are content. True or false? Glendale Company traded land with a value of $29,500 and a tax basis of $18,000 for land in a different location valued at $27,500. To equate the value of the property exchanged, the other party also paid Glendale $2,000 cash. All Like kind exchange criteria was met. What is Glendale's recognized gain on the exchange? None of the choices are correct. a. $2,000 b. $0. c. $29,500 d. $27,500 REQUIRED: Prepare a reflection paper that answers the followingindependent questions:1. Assume that you are forming a partnership with some of yourclose friends. What stipulation/s would you like t 1. FGH Inc., bought a truck for 1,500,000 php with an estimated life of 8 years. The trade-in value of the truck is130,000 php. Determine the annual depreciation & book value of the truck at the end of 4 years using: a.) SLM b.) SYDM (Ocampo's Formula) c.) DBM (Matheson's Formula). In our country, only state governments use initiatives andreferendums (I&R) to measure the issues relevant to theircitizens. Recent ballot initiatives included same-sex marriage,marijuana legal 1.What will be the increase in frequency if these waves are reflected from a 95.0mi/h fastball headed straight toward the gun? (Note: 1 mi/h = 0.447 m/s )??? (a) Suppose we have preferences U(X, Y) = min [2X, Y]. Graph/sketch the indifference curvethrough the bundle X = 10 and Y = 10. What is the utility at (10, 10)? Explain why theindifference curve looks the way it does.(b) What do we mean by a composite good? What does this composite good look like with thesepreferences? Show and explain.(c) Let Px = $10, Py = $20 and income M = $600. State the consumers maximization problem andexpress this in words.(d) Find optimal X, Y, and the resulting Utility. Explain how you get the optimum and show/sketchin a figure.(e) Now suppose we offered a discount so that good Y was priced at $5 for the first 10 units but risesto $10 for any quantity above that. Draw the new budget line.(f) Find the optimal X, Y and the resulting Utility given the availability of the discount. Compare tothe non-discounted case and discuss why it is different/the same. write the complete electron configuration for bromine using the periodic table Find the surface area of the part of the sphere x2+y2+z2=64 that lies above the cone z=(x2+y2).